Wednesday, October 29, 2008

Pre 9 Dysrhy/CAD

Which of the following generalized statements about antidysrhythmic drugs is true?

A. They increase automaticity, speed conduction rates, and increase
refractoriness.
B. They cause impotence and poor pregnancy outcomes.
C. They cause dysrhythmia.
D. They cause hypertension and tachycardia.



The electrophysiological basis of automaticity is

A. rapid influx of calcium.
B. inhibition of the sodium-potassium pump.
C. slow leak of sodium into the cell.
D. rapid influx of potassium.



One strategy for treating angina is to reduce heart work. Which of the following drugs would be used to reduce heart work?

A. diltiazem (CARDIZEM)
B. digoxin (LANOXIN)
C. epinephrine (ADRENALINE)
D. dobutamine (DOBUTREX)



Most antidysrhythmic drugs act by:

A. antagonism of receptors on the SA and AV node
B. promoting metabolism of normal neurotransmitters
C. creating retrograde electrical flow
D. blocking ion channels in myocardial cells



An action potential characterized by a slow leak of ions into the cell, that has a phase 0, phase 3, and phase 4, is an __________.

A. fast action potential.
B. muscle action potential
C. slow action potential
D. repolarization potential



All of the following decrease heart work and oxygen demand EXCEPT

A. decreasing contractility.
B. decreasing heart rate.
C. increasing duration of diastole.
D. increasing preload.



When looking at an electrical tracing of the heart, the passage of electricity through the atria is displayed as:

A. The QRS
B. The AV node
C. The QT interval
D. The P wave



Which of the following should be given in the immediate time period after a heart attack?

A. beta blockers and aspirin
B. amiodarone and hydrocortisone
C. calcium channel blockers and acetaminophen
D. quinidine and lidocaine



An action potential characterized by a rapid influx of sodium through sodium channels in phase 0 and delayed repolarization secondary to calcium influx is an __________.

A. fast action potential
B. muscle action potential
C. slow action potential
D. repolarization potential
4.


If it took extra long for electricity to get from the atrium to the ventricle, what would you see on the EKG?

A. A prolonged PR interval.
B. A prolonged ST segment.
C. A wide QRS.
D. An inverted P wave.
5.


Drugs useful in treating angina include:

1. Calcium channel blockers because they relax arterial smooth muscle and may slow heart rate.
2. Sodium channel blockers because they slow conduction within the heart.
3. Potassium channel blockers because they increase the refractory period.
4. Magnesium channel blockers because they reduce preload and afterload.

Post 8 HTN/ Lipids

Before starting atorvastatin (LIPITOR), it is important to:

A. Document that the patient failed therapy with Niacin and one of the bile acid
sequestrants (Colestipol or Cholestyramine).
B. Be sure the patient is not pregnant.
C. Teach the patient how to do home blood sugar monitoring and be sure patient has
the equipment.
D. Document normal conductivity through heart with EKG.



Some antihypertensive drugs block both alpha and beta receptors. Why would this be a good thing for someone with hypertension?

A. it causes vasodilation
B. it decreases renin release
C. it increases cardiac output
D. 1, 2 only
E. 1,2,3



The mechanism of action of this drug is dilation of arterioles by a direct effect on vascular smooth muscle.

A. clonidine (CATAPRES)
B. captopril (CAPOTEN)
C. hydralazine (APRESOLINE)
D. atropine



Losartan (COZAAR) is an angiotensin II receptor antagonist which would be expected to have which side effect(s):

A. Hypokalemia (low potassium)
B. Pupil constriction
C. Dizziness
D. Dry mouth



Aldosterone alters blood pressure because it

A. causes potassium excretion.
B. dilates blood vessles.
C. causes sodium retention.
D. enhances renin release.



In acute head injury, the osmotic diuretic mannitol may be given to reduce brain swelling. What potential problem would you need to monitor for?

A. The mannitol prevents electrolytes from being excreted by the
kidney.
B. The blood pressure could drop quickly as fluid is pulled from cells into the
blood stream.
C. Urine output typically drops transiently during the first 4 hours.
D. Over time, if the mannitol diffuses into the brain, it would make the swelling
worse.



All of the following are adverse effects of furosemide (LASIX) EXCEPT

A. high blood potassium levels
B. low blood sodium and chloride levels
C. hypotension
D. hearing loss



Hyperlipidemia drug therapy for a person with existing coronary heart disease should begin when:

A. LDL cholesterol is greater than 100.
B. HDL cholesterol is elevated.
C. The patient has lost 10% of his/her body weight.
D. Blood pressure is under control.



R.J. is a 55 yo male with long standing hypertension who has been treated with Dyazide one capsule each day. On this clinic visit, a urine sample is positive for spilled protein. His BP is 136/82. You should:

A. Discontinue the Dyazide. After 1 week, start captopril 25 mg twice each
day.
B. No drug changes are required. He should have a low protein diet.
C. Increase the Dyazide dose to 2 capsule each day.
D. Add captopril 50 mg three times each day.



P.P. is a 52 yo diabetic AA female who just had a stroke. She has had long term HTN. Current meds are: lisinopril 20 mg each day, labetalol 400 mg twice a day, lasix 20 mg twice each day. Current labs include sodium 138 (normal 135-145) and potassium 3.5 (normal 3.5-5). Her BP is 210/118 and her pulse is 75. You should:

A. Increase the labetalol.
B. Add losartan.
C. Increase the lasix.
D. Increase the lisinopril.



A patient crushed his sustained-release verapamil tablet before ingesting it. Which action would you most likely take?

A. Administer short acting verapamil to keep the heart rate down.
B. Check heart rate and administer a beta-blocker such as atenolol (TENORMIN) to
keep the heart rate down.
C. Check the serum creatinine and start dialysis.
D. Check the BP and give fluids and other medication as necessary to keep the BP
up.



P.J. is a 56 year old male who is taking an angiotensin receptor blocker for hypertension. You should teach him to

A. discontinue his medications if he develops a headache.
B. take his medication first thing in the morning to counteract the
sedation.
C. purchase his new prescription 3-4 days before his old prescription runs
out.
D. increase his salt intake to counteract the salt wasting associated with this
drug.



A drug that antagonizes aldosterone would be useful in treating hypertension because it would

A. cause potassium excretion.
B. enhance renin release.
C. block vasoconstriction
D. increase renal excretion of sodium.



Which assessment finding best reflects effective management of hypertension?

A. blood pressure 155/95
B. blood pressure reduced 10%
C. blood pressure less than 130/90
D. increased urinary output with edema reduction


A patient without other cardiac risks has triglycerides 300 mg/dl, HDLs 40 mg/dl and LDLs 150 mg/dl. Which drug (if any) should be given?

A. fenofibrate
B. losartan
C. cholestyramine
D. atorvastatin



John Doe is taking spironolactone (ALDACTONE). You should teach him to

A. take this medication in the evening to avoid sedation.
B. take potassium supplements as this drug lowers potassium levels in the blood.
C. take over the counter medications to reduce hair loss.
D. restrict water intake as this medication causes water retention.



.B. is a 29 yo male with HTN. He takes Hydrochlorothiazide 50 mg and lisinopril 40 mg each day. Current BP is 164/98. You should:

1. Discontinue the ACE inhibitor and add an Angiotensin Receptor Blocker.
2. Increase the ACEi dose.
3. Add a calcium channel blocker.
4. Discontinue the diuretic and add a beta blocker.


Thanks Lily Sok for the additional questions!


Hypertension results from an increase in vascular resistance, and several factors contribute to this. Which of the following would be the MOST POWERFUL way to decrease blood pressure?

A. Decrease the viscosity of the blood
B. Increase the viscosity of the blood
C. Decrease the radius of the vessel
D. Increase the radius of the vessel



Which of the following drugs is most likely to raise serum LDL cholesterol?

A. atorvastatin
B. hydrochlorothiazide
C. Niacin (Vitamin B3)
D. Cholestyramine



S.Q. is a 45 yo diabetic with mild renal disease (creatinine 1.7, normal is 0.6-1.2). She is taking valsartan 320 mg each day, hydrochlorothiazide 50 mg each day, and metoprolol 50 mg twice each day. She has tolerated these drugs without significant side effects. Today her BP is 138/94. You should:

A. Add lisinopril.
B. Increase the metoprolol.
C. Continue the medications as listed.
D. Reduce the valsartan dose.



Which of the following drugs blocks both alpha and beta receptors?

A. carvedilol (COREG)
B. clonidine (CATAPRES) Student Response
C. hydralazine (APRESOLINE) Student Response
D. propranolol (INDEROL) Student Response



All of the following are adverse effects of hydrochlorothiazide (ESIDRIX) EXCEPT

A. dizziness and faintness.
B. hypoglycemia. Student Response
C. high calcium levels.
D. paresthesia.



Diuretics are useful in treating hypertension becasue they

A. decrease preload.
B. increase vessel diameter.
C. block renin release.
D. block angiotensin converting enzyme.



The mechanism of action of furosemide (LASIX) is

A. antagonizing aldoseterone.
B. inhibiting sodium and chloride reabsorption in the loop of Henle.
C. increasing the osmolality of urine, thus increasing urinary output.
D. supressing renin release.

Wednesday, October 22, 2008

Pre 8 HTN/Lipids

The diuretic furosemide (LASIX), causes:
A. Hypernatremia (high Na+ level in blood)
B. Hyperkalemia (high K+ level in blood)
C. Hypokalemia (low K+ level in blood)
D. Edema and hypertension


Enalapril (VASOTEC) is an example of an antihypertensive medicne from which class?
A. ACE inhibitor
B. Angiotensin II Receptor Antagonist
C. Vasodilator
D. Beta Blocker


Which is the best approach to treating hard to control hypertension?

A. start with one drug at the normal dose and increase the dose until it
works.
B. use the maximum dose of one drug and add a second drug at its maximum
dose.
C. use the maximum dose and add a small amount of an antagonist.
D. start with a low dose of one drug and increase the dose slowly. It it is not
effective change drugs or add a second drug at a low dose.


What lab work should be done to monitor a patient's response to atorvastatin (LIPITOR)?
A. Blood sugar and Hemoglobin A1C
B. Potassium level
C. Total Cholesterol, HDL cholesterol, LDL cholesterol
D. Drug blood levels

A patient has a BP of 138/84. What condition would indicate a need to treat this patient with hypertensive medication?
1. Diabetes
2. High HDL cholesterol level
3. Age over 60 years.
4. Female gender.


Most patients with a blood pressure breater than 160/100 should be treated with:
1. A thiazide diuretic and one drug from another class.
2. Two beta blockers.
3. Lifestyle modification only.
4. A calcium channel blocker and an angiotensin receptor blocker.


All of the following drugs classes are used to treat hypertension EXCEPT
A. alpha-2 agonists
B. beta-2 agonists
C. vasodilators
D. calcium channel blockers


Enalapril (VASOTEC) treats hypertension by
A. Stimulating alpha1 receptors on blood vessels.
B. supressing renal excretion of sodium.
C. depressing contractility of the heart.
D. inhibiting angiotensin converting enzyme.


Additional Questions courtesy of Carissa Galvan, and Lily Sok, Thanks!!!


Teaching for a person taking an ACE inhibitor should include

A. report feelings of faintness to the health care provider.
B. if there is difficulty breathing or hoarsness, go to bed immediately.
C. monitor blood pressure at least once a year.
D. If you had angioedema from prior ACE inhibitor use, take a tylenol prior to taking your ACE inhibitor.


The serious, potentially life-threatening adverse of atorvastatin (LIPITOR), particularly if it is combined with a fibrate is

A. hyperglycemia
B. gastrointestinal bleeding.
C. chest pain.
D. skeletal muscle damage (rhabdomyolysis).



The Angiotensin Converting Enzyme (ACE) Inhibitor, ekanioril (VASOTEC) is associated with which adverse reaction(s) in as many as 1/3 of those who take it?

A. Hypokalemia (low K+ in blood)
B. Paradoxical hypertension
C. Liver failure
D. Cough and rash



A patient has a BP of 138/84. What condition would indicate a need to treat this patient with hypertensive medication?

1. Diabetes
2. High HDL cholesterol level
3. Age over 60 years.
4. Female gender.

Pre 7 ANS2/CHF

Quiz 7 courtesy of Lily Sok, Jenny Lou, and Carissa Galvan. Thanks you very much ladies, we really appreciate it!!!

Beta-adrenergic blocking agents are used in the management of
A. asthma.
B. cirrhosis.
C. heart failure.
D. pneumonia.


Sympathomimetic drugs are likely to cause which of the following adverse effects?
A. bradycardia
B. tachycardia
C. hypertension
D. hyperglycemia
E. 1, 3, 4 only
F. 2, 3, 4 only


The adverse effects of beta blockers include
A. increased heart rate.
B. hypertension
C. bronchoconstriction.
D. urinary urgency.


A parasympathomimetic drug such as physostigmine (ANTILERIUM) is used to
1. decrease saliva production.
2. decrease motion sickness.
3. treat urinary retention.
4. counteract adverse effects of polocarpine.


Scopolamine may be used to treat motion sickness. Side effects of dry mouth, blurred vision and constipation result from
A. Inhibition of nicotinic receptors.
B. Stimulation of alpha receptors.
C. Inhibition of muscarinic receptors
D. Inhibition of dopamine recpetors


Drugs such as an alpha-1 agonist,that mimic the sympathetic nervous system would cause
A. pupil dilation.
B. decreased heart rate.
C. consriction of airways.
D. diarrhea.


The effects of sympathomimetic drugs include
A. decreased heart rate.
B. decreased blood sugar.
C. bronchoconstriction.
D. urinary retention.


Drug Y is an adrengeric receptor blocker. This means it
1. blocks acetylcholine receptors.
2. blocks nicotinic receptors.
3. simulates dopamine receptors.
4. blocks norepinephrine receptors.


More question courtesy of Corina Kendrick, Thanks!!!



Phenylephrine (NEOSYNEPHRINE)is an alpha-1 agonist. A predictable adverse effect of this drug is
A. hypertension.
B. Myasthenia Gravis.
C. sedation.
D. heart failure.


Which drug is administered parenterally in emergency situations, such as treating acute, liefe-threatening allergic reactions?
A. dobutamine (DOBUTREX)
B. dopamine (INTROPIN)
C. epinephrine (ADRENALIN)
D. norepinephrine (LEVOPHED


Phenylephrine (NEOSYNEPHRINE)is an alpha-1 agonist. This drug would be used to treat
A. hypertension.
B. Myasthenia Gravis.
C. nasal congestion.
D. heart failure.

Saturday, October 18, 2008

Post 7 ANS2/CHF

The mechanism of action of atropine is
A. activation of post-synaptic muscarinic receptors.
B. blockade of pre-synaptic alpha receptors.
C. antagonism of acetylcholine binding sites.
D. releasing norepinephrine from nerve endings.


Your patient has high blood pressure and you are considering giving him Beta-adrenergic blocking agents. Which of the following questions should you ask prior to ordering a beta blocker like propranolol (INDERAL)?
A. Do you have asthma?
B. Do you have food intolerances?
C. Do you have a family history of alcholism?
D. Are you allergic to penicillin?


Which of the following drugs would be effective in treating hypertension?
A. albuterol (PROVENTIL)
B. DOBUTamine (DOBUTREX)
C. phenylephrine (NEOSYNEPHRINE)
D. prazosin (MINIPRESS)


You are making a home visit for physical therapy on a patient with a history of hypertension. When you ask if there have been any changes since your last visit, the patient tells you that yesterday the doctor increased the prazosin (MINIPRESS) dose. This alerts you to check for:

A. Anaphylactic allergic reaction.
B. wheezing
C. Hypotension and dizziness when the patient stands up.
D. Muscle spasms and tremors


The patient is in mild to moderate heart failure but is stable and is seen in the outpatient clinic. The heart rate is slightly elevated at 90. BP is normal at 110/70. The patient receives carvedilol (COREG) to:

A. Suppress the action of acetylcholine at muscarinic receptors thereby decreasing heart rate.
B. Activate alpha-2 receptors thereby decreasing SNS outflow from the central nervous system
C. Reduce afterload so the heart doesn't have to work so hard.
D. Stimulate alpha-1 receptors and keep the blood pressure up.


Your patient came in with the following symptoms: fever, flushed dry skin, confusion, blurred vision, and constipation. If you suspect atropine overdose, what should you give this patient?
A. atropine
B. physostigmin (ANTILERIUM)
C. bethanichol
D. pilocarpine


Pseudoephedrine is an alpha agonist added to many over the counter medications to treat:
A. diabetes
B. nasal congestion
C. insomnia
D. diarrhea


If a person has heart failure, the cardiac output is lower than is needed for adequate tissue blood flow. How would you improve cardiac output in a person who has lung congestion, jugular vein distention, tachycardia, and ankle edema?
A. decrease afterload.
B. increase increase preload.
C. increase heart work.


Drugs that improve morbidity and mortality in heart failure include:
A. Digoxin
B. Phenytoin (Dilantin)
C. Calcium channel blockers such as diltiazem (CARDIZEM)
D. The beta blocker carvedilol


A patient taking spironolactone (ALDACTONE) should be taught about adverse effects such as
A. alkalosis (high pH).
B. high potassium levels in the blood.
C. nausea.
D. constipation.


The mechanism of action of a direct acting beta agonist is
A. activation of post-synaptic receptors.
B. blockade of pre-synaptic receptors.
C. antagonism of acetylcholine binding sites.
D. releasing norepinephrine from nerve endings.


Beta-adrenergic blocking agents are used in the management of
A. asthma.
B. cirrhosis.
C. heart failure.
D. pneumonia.


A pregnant asthmatic patient leaves home for the birth clinic with the contractions are 10 minutes apart. She takes a dose of albuterol (PROVENTIL) before leaving home. What effect would you anticipate the albuterol would have?

A. Labor will be slowed as B-2 agonists relax the uterus and dilate the airways.
B. The fetus may undergo oxygen loss due to vasoconstriction associated with SNS activation.
C. The blood sugar may drop suddenly.
D. All of the above.


Propranolol (INDERAL) adverse effects include all of the following EXCEPT
A. bronchoconstriction.
B. hypertension.
C. impotence.
D. fatigue.


Drugs that improve morbidity and mortality in heart failure include:
A. Digoxin
B. Potassium supplements
C. Calcium channel blockers
D. ACE inhibitors


Patients who have NYHA class III heart failure should be taking
1. beta 2 adrenergics such as albuterol
2. non-selective beta blockers such as carvedilol.
3. peripheral alpha blockers such as prazocin
4. angiotensin converting enzymes such as angiotensin


Thanks Carissa for these additional questions!!

The mechanism of action of amphetamines is
A. interaction with post-synaptic receptors.
B. blockade of pre-ganglionic nicotinic receptors.
C. antagonism of acetylcholine binding sites.
D. releasing norepinephrine from nerve endings.


Which of the following drug classes produces the desired pharmacological effects by interacting with presynaptic receptors to reduce SNS outflow from the CNS?
A. alpha 1 agonists
B. alpha 2 agonists
C. beta 1 agonists
D. beta 2 agonists


Thanks Lily for the additional questions.


Drugs that improve morbidity and mortality in heart failure include:

A. Digoxin
B. Potassium supplements
C. ACE inhibitors Student Response
D. Calcium channel blockers



Atropine has which pharmacological property?
A. dilation of pupil
B. excessive sweating
C. decreased heart rate
D. increased GI motility

Monday, October 13, 2008

Post 6 ANS 1

The antidote for atropine poisoning is

A. bethanechol (URECHOLINE)
B. phenytoin (DILANTIN)
C. physostigmine (ANTILIRIUM)
D. propranolol (INDERAL)


The antidote for overdose with a parasympathomimetic drug is:

A. atropine.
B. physostigmine (ANTILIRIUM).
C. edrophonium chloride (TENSILON)
D. propranolol (INDERAL)


Mr. Jones is a 75 year old man who was admitted to the hospital because he fainted at home. His heart rate was only 40 and his BP was low. He is given atropine. What side effects do you need to observe for?

A. Urinary retention
B. Diarrhea
C. Diaphoresis (sweating)
D. Heart block or other very slow heart rate














Extravasation of an alpha1 agonist (such as epinephrine) into the tissue surrounding a vein (or infused accidentally into an artery) can cause such severe vasoconstriction as to cut off blood supply and cause death of the tissue. What drug could be given to block this vasocontriction?

A. Phentolamine
B. Clonidine
C. Atenolol
D. Ephedra


Your patient came in with the following symptoms: fever, flushed dry skin, confusion, blurred vision, and constipation. If you suspect atropine overdose, what should you give this patient?

A. atropine
B. physostigmin (ANTILERIUM)
C. bethanichol
D. pilocarpine


Which of the following is a predictable adverse effect of clonidine (CATAPRES), an alpha-2 agonist?

A. insomnia
B. hypotension and dizziness.
C. diarrhea
D. bronchospasm


Atropine has which pharmacological property?

A. constriction of pupil
B. excessive sweating
C. increased blood sugar
D. constipation and decreased GI motility

Knowing herself to be prone to motion sickness, before her trip to the mountains, Dizzy Lizzy places a transdermal scopolamine patch behind her ear. The transdermal application will allow blockage of the muscarinic receptors in the pathway from the inner ear to the vomiting center of the brain. Although fairly selective for these receptors, there is some action at other muscarinic sites resulting in side effects of:

A. dry skin and mouth
B. diarrhea and uterine cramping
C. CNS stimulation: insomnia and seizures
D. bradycardia and urinary frequency


For many years, patients with psychosis were treated with phenothiazines like chlorproMAZINE (Thorazine). Troubling side effects included anticholinergic problems such as:

A. Constipation
B. Insomnia
C. Urinary frequency
D. Hypertension


A patient developed urinary retention after surgery. She was prescribed bethanechol (UROCHOLINE). Why is it important that nothing be obstructing the urethra?

A. The drug works by stimulating receptor sites on the urethra. Obstruction
prevents this stimulation.
B. Obstruction increases the risk of incontinence from this drug.
C. Skeletal muscles must first be in the relaxed state to allow this drug to
work.
D. Bladder contraction against an obstruction could cause backflow of urine
into the kidneys.


K. Samuels was admitted to the emergency department with the following symptoms: excessive saliva production, copious tear production, frequent urination, diarrhea, anxiety, and hypotension. K. Samuels' symptoms are characteristic of __________ drug overdose.

A. parasympathomimetic/cholinergic
B. sympathomimetic/adrengergic
C. parasympatholytic
D. sympatholytic


Mr. Jones is a 75 year old man who was admitted to the hospital because he fainted at home. His heart rate was only 40 and his BP was low. What drug should be given?

A. The alpha-2 agonist clonidine.
B. The parasympathomimetic drug neostigmine.
C. The anticholinergic drug atopine.
D. The beta-2 agonist albuterol


Phenylephrine (NEOSYNEPHRINE)is an alpha-1 agonist. A predictable adverse effect of this drug is

A. hypertension.
B. Myasthenia Gravis.
C. sedation.
D. heart failure.


Which of the following drug classes produces the desired pharmacological effects by interacting with presynaptic receptors to reduce SNS outflow from the CNS?

A. alpha 1 agonists
B. alpha 2 agonists
C. beta 1 agonists
D. beta 2 agonists


Atropine has which pharmacological effect?

A. dilation of pupil
B. excessive sweating
C. decreased heart rate
D. increased GI motility


For many years, patients with psychosis were treated with phenothiazines like chlorproMAZINE (Thorazine). Troubling side effects included anticholinergic problems such as:

A. Hypertension
B. Diarrhea
C. Urinary retention
D. Cough


A patient developed urinary retention after surgery. She was prescribed bethanechol (UROCHOLINE). Adverse effects of this drug include:

A. abdominal cramps, hypotension, excess salivation
B. Pupil dilation, tachycardia, constipation
C. Dry as a bone, red as a beet, blind as a bat, hot as a furnance, mad as a
hatter
D. Hypertension, tachycardia, increased urine output,
hyperglycemia



A patient receives atropine preoperatively. Why?

A. To prevent them from having to urinate during the surgery.
B. To decrease respiratory secretions and keep heart rate up.
C. To temporarily paralyze muscles so the patient holds still.
D. To prevent infection.



F. Ames presented with the following symptoms: dry skin, facial flushing, visual problems, elevated temperature, confusion, tachycardia, urinary retention, and constipation. Which of the following drugs should be administered?

A. phentolamine (REGITINE)
B. physostigmine (ANTILERIUM)
C. atropine
D. propranolol (INDEROL)
6.


Which of the following is a predictable adverse effect of clonidine (CATAPRES), an alpha-2 agonist?

A. drowsiness
B. hypertension
C. diarrhea
D. bronchospasm



Edrophonium (Tensilon) is a short-acting AchE inhibitor used to diagnose myasthenia gravis. In a patient with MG, this drug should:

A. Relax muscle spasm.
B. Raise the blood pressure.
C. Improve muscle strength.
D. Increase heart rate.

Monday, October 6, 2008

Pre 6 ANS 1

Which of the following components of the nervous system has a short preganglionic fiber, and long post-ganglionic fiber?

A. ANS
B. Parasympathetic Nervous System
C. Sympathetic Nervous System
D. Somatic Nervous System


The neurotransmitter at adrenergic nerve endings is

A. acetylcholine.
B. norepinephrine.


The neurotransmitter at SNS ganglia is

A. acteylcholine.
B. dopamine.
C. norepinephrine.
D. substance P.


The recptor types occupied by norepinephrine include

A. alpha.
B. beta.
C. muscarinic.
D. nicotinic.
E. 1 and 2


Your patient is scheduled for a dilated eye exam. She is given atropine-like eye drops to dilate the pupils. You should teach her to

1. avoid driving and hazardous activities until she regains ability to focus.
2. take her heart rate every hour for two days.
3. suck on ice chips to ensure hydration.
4. take a laxative for the next two days to prevent constipation.


Which of the following components of the nervous system is specifically responsible for "fight or flight" responses?

A. ANS
B. Parasympathetic Nervous System
C. Sympathetic Nervous System
D. Somatic Nervous System


The neurotransmitter at cholinergic nerve endings is

A. acetylcholine.
B. norepinephrine.


The neurotransmitter at PNS ganglia is

A. acteylcholine.
B. dopamine.
C. norepinephrine.
D. substance P.


The recptor types occupied by acetylcholine include

A. alpha.
B. beta.
C. muscarinic.
D. nicotinic.
E. 3 and 4


Thanks Lily for the extra questions, we appreciate it!!!

Which of the following components of the nervous system is specifically responsible for vegitative, resting functions?

A. ANS
B. Parasympathetic Nervous System
C. Sympathetic Nervous System
D. Somatic Nervous System
5.


Norepinephrine is inactivated by MAO and COMT. What do you expect would happen if you blocked these two enzymes?

A. less SNS activity
B. more SNS activity
C. nothing would happen as there are other ways to break down norepinephrine.



The recptor types occupied by norepinephrine include

A. alpha.
B. beta.
C. muscarinic.
D. nicotinic.
E. 1 and 2

Post 5 Vac/Coag

Common side effects from vaccines include

A. slight fever and pain at the injection site.
B. seizures.
C. generalized pain.
D. vomiting.



A patient complained of weakness on one side of the body and had trouble speaking. They are seen 30 minutes later in the ER. Brain scan confirms a non-hemorragic stoke. What drug would be the most likely choice in the Emergency Room to restore blood flow to the brain?

A. Tissue Plasminogen Activator (TPA)
B. Coumadin (WARFARIN)
C. Heparin
D. Atorvastatin (LIPITOR)



One day after knee surgery, a patient is started on both heparin and coumadin. The rationale for doing this is:

A. Heparin works right away and provides anticoagulation until the coumadin starts working a few days later.
B. Coumadin works right away and provides anticoagulation until the heparin starts working a few days later.
C. Coumadin will reduce the risk of bleeding from the heparin.
D. Heparin will reduce the risk of bleeding from the coumadin.



A patient taking clopidogrel (Plavix) should have which lab test done to monitor for adverse effects?

A. INR
B. aPTT
C. Hematocrit
D. Platelet



Your 80 year old patient tells you he takes the herb ginko to "improve his aging memory". You are alert to the potential for drug interaction with which of his other medications?

A. digoxin
B. furosemide (LASIX)
C. potassium
D. warfarin (COUMADIN)



Which of the following is an example of passive immunization?

A. An elderly patient receives an influenza vaccination every year which causes the person to produce antibodies against influenza virus.
B. A child receives the inactivated polio vaccine instead of the live attenuated form.
C. An unborn baby receives antibodies from the mom across the placenta.
D. The World Health Organization uses oral polio vaccine rather than one that is injected with a needle.



Which of the following is an example of active immunization?

A. An elderly patient receives an influenza vaccination every year which causes the person to produce antibodies against influenza virus.
B. An infant receives respiratory syncytial virus intravenous immune globulin (RespiGam) every month during winter and spring.
C. An unborn baby receives antibodies from the mom across the placenta.
D. A person receives rabies immune globulin after being bitten by an infected animal.



A healthy, low-risk child who has received all scheduled immunization comes in for a 5 year old, preschool check up. What immunizations will be given?

A. Boosters for DTaP, IPV, varicella and MMR
B. New immunization series for MMR and Varicella
C. New immunization series for H. Influenza, polio, and pneumococcal
D. Boosters for Hepatitis B and tetanus



A healthy 47 year old who lives with his healthy wife and works in a library, should receive which vaccination?

A. pneumococcal pneumonia if not received within the last 5 years
B. zoster if antibody titer to chickenpox is low
C. influenza if not received within the within the last 10 years
D. tetanus toxoid if not received within the last 10 years



One of your friends tells you that giving a child multiple vaccinations for different diseases at the same time increases the risk of harmful side effects and can overload the immune system. Your response to this friend should be:

A. You are correct, but only for infants. Very young children (<1 year) cannot make antibodies against more than one antigen at a time.
B. You are correct, but only for older children. Children 6-12 years of age understand the concept of getting a shot to prevent an infection. Psychologically, they do better with one shot for one disease for one visit.
C. You are correct but only for adolescents. It is not until puberty that vaccine boosters work better when separated from other vaccines by at least one month..
D. There have been many studies showing vaccine efficacy when given in combination. Parents save time and money with fewer office visits, which might improve compliance. Kids prefer fewer trips to the clinic and fewer shots too.



Lab tests are done to monitor anticoagulant dosing. For which patient would you hold the heparin?

A. The aPTT is 100 (normal is 25-40 seconds)
B. The INR is 1.8 (normal is 1)
C. The aPTT is 20 (normal is 25-40 seconds).
D. The INR is 0.8 (normal is 1)



A 56 yo female (whose last pertussis vaccination was as a child), has a 3 yo child with whooping cough. She asks about spread of infection to herself and her 4 yo son who has received the following vaccinations: 1 MMR, 4 Hib, 3 hepB, 3 IPV and 4 DTaP. You should respond:

1. Your son should be ok, but watch for onset of persistant cough in yourself.
2. You and your son should both get boosters.
3. You and your 4 yo should be protected, but do wash your hands well after caring for your younger child.
4. You should be ok, but your 4 yo should receive pneumococcal conjugated vaccine.



At the 2 month checkup, an infant should receive (along with other vaccines not listed):

A. Measles
B. Rotavirus
C. dTP
D. Hepatitis A



Thanks Lily for the additional questions!!!


A 60 year old male has risk factors for a myocardial infarction (heart attack) because he smokes, has high blood pressure and has several family members who had heart attacks. What drug should be given to prevent a heart attack?

A. Tissue Plasminogen Activator (TPA)
B. Coumadin (WARFARIN)
C. Heparin
D. Aspirin
7.


Your 65 year old patient has chronic atrial fibrillation of the heart and is at risk for throwing a blood clot to the brain. What drug would be the best choice to prevent blood clot development in this patient?

A. Tissue Plasminogen Activator (TPA)
B. Coumadin (WARFARIN)
C. Heparin
D. Atorvastatin (LIPITOR)
8.


Lab tests are done to monitor anticoagulant dosing. For which patient would you hold the coumadin?

A. The aPTT is 30 (normal is 25-40 seconds).
B. The aPTT is 60 (normal is 25-40 seconds)
C. The INR is 0.8 (normal is 1)
D. The INR is 5 (normal is 1)



An elderly person residing in a skilled nursing facility needs:

A. Tetanus and diptheria toxoids every 10 years.
B. Human Papilloma virus once at age 70.
C. Varicella if the titer of antibodies is high
D. Meningococcocal once at age 80